Difference between revisions of "2009 AIME II Problems/Problem 15"
(→Solution) |
|||
Line 5: | Line 5: | ||
(For some reason, I can't submit LaTeX for this page.) | (For some reason, I can't submit LaTeX for this page.) | ||
− | Let O be the center of the circle. Define <MOC = t, <BOA = 2a, and let BC and AC intersect MN at points X and Y, respectively. We will express the length of XY as a function of t and maximize that function in the interval [0, pi]. | + | Let <math>O</math> be the center of the circle. Define <math>\angle{MOC}=t</math>, <math>\angle{BOA}=2a</math>, and let <math>BC</math> and <math>AC</math> intersect <math>MN</math> at points <math>X</math> and <math>Y</math>, respectively. We will express the length of <math>XY</math> as a function of <math>t</math> and maximize that function in the interval <math>[0, \pi]</math>. |
− | Let C' be the foot of the perpendicular from C to MN. We compute XY as follows. | + | Let <math>C'</math> be the foot of the perpendicular from <math>C</math> to <math>MN</math>. We compute <math>XY</math> as follows. |
− | (a) By the Extended Law of Sines in triangle ABC, we have | + | (a) By the Extended Law of Sines in triangle <math>ABC</math>, we have |
− | CA | + | <cmath>CA</cmath> |
− | = sin< | + | <cmath>= \sin\angle{ABC}</cmath> |
− | = sin( | + | <cmath>= \sin\left(\frac{\widehat{AN} + \widehat{NC}}{2}\right)</cmath> |
− | = sin( | + | <cmath>= \sin\left(\frac{\frac{\pi}{2} + (\pi-t)}{2}\right)</cmath> |
− | = sin( | + | <cmath>= \sin\left(\frac{3\pi}{4} - \frac{t}{2}\right)</cmath> |
− | = sin(pi | + | <cmath>= \sin\left(\frac{\pi}{4} + \frac{t}{2}\right)</cmath> |
− | (b) Note that CC' = | + | (b) Note that <math>CC' = CO\sin(t) = \left(\frac{1}{2}\right)\sin(t)</math> and <math>AO = \frac{1}{2}</math>. Since <math>CC'Y</math> and <math>AOY</math> are similar right triangles, we have <math>CY/AY = CC'/AO = \sin(t)</math>, and hence, |
− | CY/CA | + | <cmath>CY/CA</cmath> |
− | = CY | + | <cmath>= \frac{CY}{CY + AY}</cmath> |
− | = sin(t) | + | <cmath>= \frac{\sin(t)}{1 + \sin(t)}</cmath> |
− | = sin(t) | + | <cmath>= \frac{\sin(t)}{\sin\left(\frac{\pi}{2}\right) + \sin(t)}</cmath> |
− | = sin(t) | + | <cmath>= \frac{\sin(t)}{2\sin\left(\frac{\pi}{4} + \frac{t}{2}\right)\cos\left(\frac{\pi}{4} - \frac{t}{2}\right)}</cmath> |
− | (c) We have <XCY = | + | (c) We have <math>\angle{XCY} = \frac{\widehat{AB}}{2}=a</math> and <math>\angle{CXY} = \frac{\widehat{MB}+\widehat{CN}}{2} = \frac{\left(\frac{\pi}{2} - 2a\right) + (\pi - t)}{2} = \frac{3\pi}{4} - a - \frac{t}{2}</math>, and hence by the Law of Sines, |
− | XY/CY | + | <cmath>XY/CY</cmath> |
− | = sin | + | <cmath>= \frac{\sin\angle{XCY}}{\sin\angle{CXY}}</cmath> |
− | = sin(a) | + | <cmath>= \frac{\sin(a)}{\sin\left(\frac{3\pi}{4} - a - \frac{t}{2}\right)}</cmath> |
− | = sin(a) | + | <cmath>= \frac{\sin(a)}{\sin\left(\frac{\pi}{4} + a + \frac{t}{2}\right)}</cmath> |
(d) Multiplying (a), (b), and (c), we have | (d) Multiplying (a), (b), and (c), we have | ||
− | XY | + | <cmath>XY</cmath> |
− | = CA * (CY/CA) * (XY/CY) | + | <cmath>= CA * (CY/CA) * (XY/CY)</cmath> |
− | = sin(t)sin(a) | + | <cmath>= \frac{\sin(t)\sin(a)}{2\cos\left(\frac{\pi}{4} - \frac{t}{2}\right)\sin\left(\frac{\pi}{4} + a + \frac{t}{2}\right)}</cmath> |
− | = sin(t)sin(a) | + | <cmath>= \frac{\sin(t)\sin(a)}{\sin\left(\frac{\pi}{2} + a\right) + \sin(a + t)}</cmath> |
− | = sin(a) | + | <cmath>= \sin(a)\times\frac{\sin(t)}{\sin(t + a) + \cos(a)}</cmath>, |
− | which is a function of t (and the constant a). Differentiating this with respect to t yields | + | which is a function of <math>t</math> (and the constant <math>a</math>). Differentiating this with respect to <math>t</math> yields |
− | sin(a) | + | <cmath>\sin(a)\times\frac{\cos(t)(\sin(t + a) + \cos(a)) - \sin(t)\cos(t + a)}{(\sin(t + a) + \cos(a))^2}</cmath>, |
and the numerator of this is | and the numerator of this is | ||
− | sin(a) | + | <cmath>\sin(a) \times(\sin(t + a)\cos(t) - \cos(t + a)\sin(t) + \cos(a)\cos(t))</cmath> |
+ | <cmath>= \sin(a) \times (\sin(a) + \cos(a)\cos(t))</cmath>, | ||
− | which vanishes when sin(a) + cos(a)cos(t) = 0. Therefore, the length of XY is maximized when t=t', where t' is the value in [0, pi] that satisfies cos(t') = -tan(a). | + | which vanishes when <math>\sin(a) + \cos(a)\cos(t) = 0</math>. Therefore, the length of <math>XY</math> is maximized when <math>t=t'</math>, where <math>t'</math> is the value in <math>[0, \pi]</math> that satisfies <math>\cos(t') = -\tan(a)</math>. |
Note that | Note that | ||
− | + | <cmath>\frac{1 - \tan(a)}{1 + \tan(a)} = \tan\left(\frac{\pi}{4} - a\right) = \tan((\widehat{MB})/2) = \tan\angle{MNB} = \frac{3}{4}</cmath>, | |
− | so tan(a) = 1/ | + | so <math>\tan(a) = \frac{1}{7}</math>. We compute |
− | sin(a) = sqrt | + | <cmath>\sin(a) = \frac{\sqrt{2}}{10}</cmath> |
− | cos(a) = | + | <cmath>\cos(a) = \frac{7\sqrt{2}}{10}</cmath> |
− | cos(t') = -tan(a) = -1/ | + | <cmath>\cos(t') = -\tan(a) = -\frac{1}{7}</cmath> |
− | sin(t') = | + | <cmath>\sin(t') = \frac{4\sqrt{3}}{7}</cmath> |
− | sin(t' + a) = sin(t')cos(a) + cos(t')sin(a) = | + | <cmath>\sin(t' + a)=\sin(t')\cos(a) + \cos(t')\sin(a) = \frac{28\sqrt{6} - \sqrt{2}}{70}</cmath>, |
− | so the maximum length of XY is sin(a) | + | so the maximum length of <math>XY</math> is <math>\sin(a)\times\frac{\sin(t')}{\sin(t' + a) + \cos(a)} = 7 - 4sqrt(3)</math>, and the answer is <math>7 + 4 + 3 = \boxed{014}</math>. |
Revision as of 21:29, 26 March 2011
Problem
Let be a diameter of a circle with diameter 1. Let and be points on one of the semicircular arcs determined by such that is the midpoint of the semicircle and . Point lies on the other semicircular arc. Let be the length of the line segment whose endpoints are the intersections of diameter with chords and . The largest possible value of can be written in the form , where and are positive integers and is not divisible by the square of any prime. Find .
Solution
(For some reason, I can't submit LaTeX for this page.)
Let be the center of the circle. Define , , and let and intersect at points and , respectively. We will express the length of as a function of and maximize that function in the interval .
Let be the foot of the perpendicular from to . We compute as follows.
(a) By the Extended Law of Sines in triangle , we have
(b) Note that and . Since and are similar right triangles, we have , and hence,
(c) We have and , and hence by the Law of Sines,
(d) Multiplying (a), (b), and (c), we have
,
which is a function of (and the constant ). Differentiating this with respect to yields
,
and the numerator of this is
,
which vanishes when . Therefore, the length of is maximized when , where is the value in that satisfies .
Note that
,
so . We compute
,
so the maximum length of is , and the answer is .